The time it takes this thing to travel across 3 points

Click For Summary
The discussion revolves around a homework problem involving a thing traveling from point P to A on land and then from A to Q in water, with specified speeds for each medium. The user seeks help in deriving the time equation T(theta) for the journey and identifying the shortest travel route. A participant suggests a correction to the time equation, indicating it should be T(theta)=π - 2θ + 5sin(θ/2). The thread emphasizes the importance of showing initial work for part (a) to facilitate assistance. The conversation highlights the need for collaboration in solving complex problems.
Mewtwo
Messages
7
Reaction score
0
Hey! I have a problem that I have no clue how to solve. So I was hoping you guys could help me. I'm not from an English speaking country so I apologize.

Homework Statement


A thing is made to travel in water and on land. In water it travels with 20 m/s speed but on land with 25m/s speed. The thing starts at point P on the edge of a pond with radius 50m.
358a23l.jpg

(I drew this picture in paint and what I forgot to include is that the curved line from P to A and the straight line from A to Q are a little bit darker then the other lines, so it seems to me that the thing travels on land from P to A but in water from A to Q).

(a) Show that the time it takes the thing to travel from P to A to Q is:
T(theta)=pi-20+5sin(theta/2)

(b) Give a description of the route that takes the shortest time for the thing to travel.

Thanks in advance!
 
Physics news on Phys.org
Mewtwo said:
(a) Show that the time it takes the thing to travel from P to A to Q is:
T(theta)=pi-20+5sin(theta/2)

I think you mean ##T(\theta)=\pi - 2\theta + 5\sin{\frac \theta 2}##. But nobody here is going to work it for you. Show us what you have for part (a.)
 
Last edited:
Question: A clock's minute hand has length 4 and its hour hand has length 3. What is the distance between the tips at the moment when it is increasing most rapidly?(Putnam Exam Question) Answer: Making assumption that both the hands moves at constant angular velocities, the answer is ## \sqrt{7} .## But don't you think this assumption is somewhat doubtful and wrong?

Similar threads

  • · Replies 10 ·
Replies
10
Views
3K
  • · Replies 8 ·
Replies
8
Views
3K
  • · Replies 10 ·
Replies
10
Views
2K
  • · Replies 14 ·
Replies
14
Views
2K
  • · Replies 2 ·
Replies
2
Views
2K
  • · Replies 21 ·
Replies
21
Views
3K
  • · Replies 16 ·
Replies
16
Views
2K
  • · Replies 25 ·
Replies
25
Views
2K
  • · Replies 3 ·
Replies
3
Views
1K
  • · Replies 2 ·
Replies
2
Views
1K